Monday 25 November 2013

discrete mathematics - Proving an inequality by mathematical induction



I'm trying to solve a problem with inequalities using mathematical induction but I am stuck halfway through the process.

The problem: Use mathematical induction to establish the inequality -
$(1 + \frac{1}{2})^n \ge 1 + \frac{n}{2}$ for n $\in \mathbb{N}$



Steps



1) $n = 1$, $(1 + \frac{1}{2})^1 \ge 1 + \frac{1}{2}$ is TRUE



2) $n = k$, assume that $(1 + \frac{1}{2})^k \ge 1 + \frac{k}{2}$ for n $\in \mathbb{N}$



3) Show the statement is true for $k + 1$




$(1 + \frac{1}{2})^{k+1}$ = $(1 + \frac{1}{2})^k * (1 + \frac{1}{2})$



$\ge$ $(1 + \frac{k}{2}) * (1 + \frac{1}{2})$ - using the assumption in step $2$



My question is, how do I continue this problem? Or did I go wrong somewhere? I just can't figure out what the next step is.


Answer



Continue with:



$(1 + \frac{k}{2}) * (1 + \frac{1}{2}) =$




$1 + \frac{k}{2} + \frac{1}{2} + \frac{k}{4} >$



$1 + \frac{k}{2} + \frac{1}{2}=$



$1 + \frac{k+1}{2}$


No comments:

Post a Comment

real analysis - How to find $lim_{hrightarrow 0}frac{sin(ha)}{h}$

How to find $\lim_{h\rightarrow 0}\frac{\sin(ha)}{h}$ without lhopital rule? I know when I use lhopital I easy get $$ \lim_{h\rightarrow 0}...